Write a polynomial function of least degree with integral coefficients that has the given zeros.
-5, -3-2i

Answers

Answer 1

Answer:

[tex]f(x)=x^3+11x^2+43x+65[/tex]

Step-by-step explanation:

If a polynomial function has a complex zero, then the conjugate of that complex zero is also a zero of the polynomial.

So, if (-3 - 2i) is a zero of the polynomial, then its conjugate (-3 + 2i) is also a zero of the polynomial.

Therefore, the three zeros of the polynomial function are:

-5(-3 - 2i)(-3 + 2i)

The zero of a polynomial f(x) is the x-value when f(x) = 0.

According to the factor theorem, if f(a) = 0 then (x - a) is a factor of the polynomial f(x).

Therefore, the polynomial function in factored form is:

[tex]\begin{aligned}f(x) &= (x - (-5))(x-(-3-2i))(x-(-3+2i))\\&= (x +5)(x+3+2i)(x+3-2i)\end{aligned}[/tex]

Expand the brackets to write the polynomial in standard form.

[tex]\begin{aligned}f(x) &=(x +5)(x+3+2i)(x+3-2i)\\&=(x+5)(x^2+3x-2xi+3x+9-6i+2ix+6i-4i^2)\\&=(x+5)(x^2+6x+9-4i^2)\\&=(x+5)(x^2+6x+9-4(-1))\\&=(x+5)(x^2+6x+9+4)\\&=(x+5)(x^2+6x+13)\\&=x^3+6x^2+13x+5x^2+30x+65\\&=x^3+11x^2+43x+65\end{aligned}[/tex]

Therefore, the polynomial function of least degree with integral coefficients that has the given zeros -5 and (-3 - 2i) is:

[tex]f(x)=x^3+11x^2+43x+65[/tex]

Answer 2

f(x)=x3+11x²+43x+65


Related Questions

Kika and Mato each took out a loan for $5,000 from the bank. Kika has an interest rate of 5. 2%, and he plans to repay the loan in 5 years. Mato has an interest rate of 7. 5%, and he plans to repay the loan in 24 months. Who will pay more in interest, and about how much more will he pay?



A:Kika; $300


B:Kika; $700


C:Mato; $700


D:Mato; $300

Answers

Mato will pay about $700 more in interest than Kika ($625 - $1,300 = $675, which rounds to $700). The answer is C: Mato; $700

Mato will pay more in interest because he has a higher interest rate and a shorter repayment period. To calculate the amount of interest each will pay, we can use the formula:

Interest = (Loan amount) x (Interest rate) x (Time in years)

For Kika:
Interest = $5,000 x 0.052 x 5
Interest = $1,300

For Mato:
Interest = $5,000 x 0.075 x (2/12)
Interest = $625

Therefore, Mato will pay about $700 more in interest than Kika ($625 - $1,300 = $675, which rounds to $700). The answer is C: Mato; $700.

To know more about  events refer here

https://brainly.com/question/12961938#

#SPJ11

Duncan's favorite park just added a statue of a badger, the state animal. The statue sits on a base shaped like a rectangular prism. The base is 5 feet long, 3 feet wide, and has a volume of 60 cubic feet. How tall is the base of the statue? Write your answer as a whole number or decimal. Do not round. PLEAS HELP â

LOL NVM

Answers

The height of the base of the statue, structured in rectangular prism shape with stated measure of dimension is 4 feet.

The volume of the rectangular prism will be given by the formula -

Volume = length × width × height

Keep the values in formula to find the value of height of the base of the statue

60 = 5 × 3 × height

Rearranging the equation in terms of height

Height = 60 × (5 × 3)

Multiplying the denominator on Right Hand Side

Height = 60/15

Divide the values

Height = 4

Hence the height is 4 feet.

Learn more about rectangular prism -

https://brainly.com/question/24284033

#SPJ4

The seventh-grade class is selling boxes of votive candles as a fundraiser. The first box purchased costs $14. 00, and each additional box costs $10. 0.


a. Is the relationship between the number of additional boxes of candles purchased and the total money spent linear?explain


b. An equation that relates the total money spent on boxes of candles, C, to the number of additional boxes purchased,b,is,


c. Using the equation from part (b), the total money spent by a person who bought 3 boxes of candles for the fundraiser would be $

Answers

No, the relationship between the number of additional boxes of candles purchased. C = 14.00 + 10.00b. The person would spend $34.00 on 3 boxes of candles.

a. No, the relationship between the number of additional boxes of candles purchased and the total money spent is not linear. This is because the cost of the first box is $14.00 and the cost of each additional box is $10.00.

A linear relationship implies that the change in the dependent variable (total money spent) is proportional to the change in the independent variable (number of additional boxes purchased), but in this case, the cost does not change at a constant rate.

b. The equation that relates the total money spent on boxes of candles, C, to the number of additional boxes purchased, b, is:

C = 14.00 + 10.00b

This equation takes into account the cost of the first box, which is $14.00, and the cost of each additional box, which is $10.00.

c. If someone buys 3 boxes of candles, they are purchasing 2 additional boxes (since the first box is already included in the $14.00). Using the equation from part (b), the total money spent by a person who bought 3 boxes of candles for the fundraiser would be:

C = 14.00 + 10.00(2) = $34.00

Therefore, the person would spend $34.00 on 3 boxes of candles.

To know more about money spent, refer here:

https://brainly.com/question/1542778#

#SPJ11

Please help asap! thank you!
solve the system of equations:
6x / 5 + y / 15 = 2.3
x / 10 - 2y / 3 = 1.2
(the slashes represent fractions.)

Answers

The solution of the given system of equations is x = 3.2 and y = 1.5.

To solve this system of equations, we can use the method of elimination, where we eliminate one of the variables by adding or subtracting the equations.

First, let's eliminate y by multiplying the first equation by 2 and the second equation by 15:

12x/5 + 2y/15 = 4.6 (multiply the first equation by 2)

3x/2 - 10y = 18 (multiply the second equation by 15)

Now we can eliminate y by multiplying the first equation by 5 and adding it to the second equation:

12x + y/5 = 23 (multiply the first equation by 5 and simplify)

12x - y = 54 (subtract the second equation from the previous equation)

Adding the two equations, we get:

24x = 77

Therefore, x = 77/24.

Substituting x = 77/24 into the first equation, we get:

6(77/24)/5 + y/15 = 2.3

Simplifying this equation, we get:

y = 1.5

To know more about system of equations, refer here:

https://brainly.com/question/21620502#

#SPJ11

At one of new york’s traffic signals, if more than 17 cars are held up at the intersection, a traffic officer will intervene and direct the traffic. the hourly traffic pattern from 12:00 p.m. to 10:00 p.m. mimics the random numbers generated between 5 and 25. (this holds true if there are no external factors such as accidents or car breakdowns.) scenario hour number of cars held up at intersection a noon−1:00 p.m. 16 b 1:00−2:00 p.m. 24 c 2:00−3:00 p.m. 6 d 3:00−4:00 p.m. 21 e 4:00−5:00 p.m. 15 f 5:00−6:00 p.m. 24 g 6:00−7:00 p.m. 9 h 7:00−8:00 p.m. 9 i 8:00−9:00 p.m. 9 based on the data in the table, what is the random variable in this scenario? a. the time interval between two red lights b. the number of traffic accidents that occur at the intersection c. the number of times a traffic officer monitors the signal d. the number of cars held up at the intersection

Answers

The random variable in this scenario is the number of cars held up at the intersection (option d).

The data provided in the table shows the number of cars held up at the intersection during specific time intervals, ranging from 12:00 p.m. to 9:00 p.m. Based on this information, it is clear that the random variable in this scenario is the number of cars held up at the intersection.

To put it in mathematical terms, let X be the random variable representing the number of cars held up at the intersection during a specific time interval. The data provided in the table represents a sample of X, with each time interval being a different observation. The values of X can range from 0 to 25, with 17 being the threshold for intervention by a traffic officer.

Therefore, the answer to the question is d. the number of cars held up at the intersection. It is important to note that this random variable is discrete, as it takes on specific integer values.

To know more about random variable here

https://brainly.com/question/17238189

#SPJ4

The three inner circles are congruent

which measurement is closest to the

area of the largest outside circle in

square centimeters?

a 56. 52 cm

b 254. 34 cm

113 04 cm

5 cm

1,017 36 cm

Answers

The area of the largest outside circle in square centimeters is closest to e)1,017.36 cm².

The area of the largest circle is equal to the sum of the areas of the three inner circles and the area of the white region between them. Since the three inner circles are congruent, we can divide the white region into three equal parts. Let the radius of each inner circle be 'r'. Then, the radius of the largest circle is '3r'.

The area of the white region is the difference between the area of the square and the sum of the areas of the three congruent sectors. The area of each sector is (1/6)πr².

Therefore, the area of the white region is (9/4) r². Finally, we can use the formula for the area of a circle to find the area of the largest circle: A = π(3r)² + 3(1/6)πr² - (9/4) r² = (63/4)πr². If we substitute the value of r as 6 cm (since the diameter of the inner circle is 12 cm), we get the area of the largest circle as (63/4)π(6)² ≈ 1,017.36 cm²(e).

For more questions like Area click the link below:

https://brainly.com/question/27683633

#SPJ11

Evaluate the following integral over the Region R. (Answer accurate to 2 decimal places). S 2x ) da 2(x + y) DA R R= {(x, y) | 9 < x² + y? < 49, x < 0} Hint: The integral and Region is defined in rectangular coordinates.

Answers

After integrating with respect to y and then x, we get the value of the integral accurate to 2 decimal places as -21.98.

First, let us express the limits of integration. Since the region R is defined in the rectangular coordinate system, we can express the limits of integration as follows:

9 < x² + y² < 49

-3 < x < 0

Next, we need to express the integral in terms of these limits of integration. The integral of 2x over the region R can be expressed as:

∫∫R 2x dA = ∫-3⁰ ∫√(9-x²)√(49-x²) 2x dy dx = -21.98

Here, we have used the fact that the region R is defined as {(x, y) | 9 < x² + y² < 49, x < 0}.

The limits of integration for y are determined by the equation of the circle centered at the origin with radius 7 and the equation of the circle centered at the origin with radius 3.

Now, we can evaluate the integral using the double integral formula.

To know more about integral here

https://brainly.com/question/18125359

#SPJ4

John bought stock for $350. A year later, he sold it for $385. What is his gain in dollars? What is his return on investment? (Round to the nearest whole percent. ) I need help, please

Answers

If John bought stock for $350 then A year later, he sold it for $385. So John's Return on investment is 10%.

To find John's gain in dollars, we need to subtract the purchase price from the selling price i.e. Gain = Selling price - The purchase price. So John's Return on investment is 10%.

Gain = $385 - $350

Gain = $35

So John's gain in dollars is $35.

To find John's return on investment (ROI), we need to use the formula:

ROI = (Gain / Investment) x 100%

We already know the gain is $35, and the investment is $350. Substituting these values into the formula, we get:

ROI = ($35 / $350) x 100%

ROI = 0.1 x 100%

ROI = 10%

So John's Return on investment is 10%. Rounded to the nearest whole percent, the answer is also 10%.

To learn more about “return on investment” refer to the https://brainly.com/question/11913993

#SPJ11

Determine whether the series n² - 5 na tn - 6 n=1 is convergent or divergent using the Limit Comparison Test.

Answers

To use the Limit Comparison Test, we need to find a series whose behavior is well-known and similar to the given series. Let's consider the series aₙ = n². We have:

limₙ→∞ (aₙ / (n² - 5naₙ - 6)) = limₙ→∞ (n² / n²) = 1

Since this limit is finite and positive, and aₙ is a convergent series (by the p-series test with p = 2), we can apply the Limit Comparison Test and conclude that the given series is convergent.
To determine if the series ∑(n² - 5n) from n=1 to infinity is convergent or divergent using the Limit Comparison Test, we need to find a comparable series and then calculate the limit of the ratio between the two series as n approaches infinity.

Let's compare the given series to a simpler series ∑n² (n=1 to infinity). Now, we'll find the limit of the ratio:

Limit (n→∞) [(n² - 5n) / n²]

As n approaches infinity, the -5n term becomes insignificant compared to the n² term. So, the limit becomes:

Limit (n→∞) [n² / n²] = 1

Since the limit is a finite, nonzero value (1 in this case), the given series and the comparison series will have the same convergence behavior. We know that the series ∑n² (n=1 to infinity) is a divergent series, as it is a p-series with p=2 (less than or equal to 1). Therefore, the given series ∑(n² - 5n) from n=1 to infinity is also divergent.

Learn more about Limit Comparison Test here: brainly.com/question/31362838

#SPJ11

"Express the volume of the part of the ball p < 5 that lies between the cones т/4 and
т/3. "

Answers

We can express the limits of integration as follows:

For z between 0 and 5/√2, x and y range from 0 to √(25 - [tex]z^2[/tex]).

For z between 5/√2 and 5/2, x and y range from 0 to √(3[tex]z^2[/tex] - 25).

For z between 5/2 and 5, x and y range from 0 to √(25 - z

Find the equation of the sphere.

The equation of a sphere with center (0,0,0) and radius r is

[tex]x^2 + y^2 + z^2 = r^2.[/tex]

In this case, we have r = 5, so the equation of the sphere is

[tex]x^2 + y^2 + z^2 = 25.[/tex]

Find the equations of the cones.

The equation of a cone with half-aperture angle θ and vertex at the origin is given by [tex]x^2 + y^2 = z^2 tan^2[/tex](θ). In this case, we have two cones: one with θ = π/4 and one with θ = π/3.

Their equations are x^[tex]2 + y^2 = z^2 tan^2(\pi /4) = z^2[/tex] and [tex]x^2 + y^2 = z^2 tan^2(\pi /3) = 3z^2.[/tex]

Find the intersection points of the sphere and the cones.

To find the intersection points, we substitute the equation of the sphere into the equations of the cones: [tex]x^2 + y^2 + z^2 = 25, x^2 + y^2 = z^2,[/tex] and x^2 + [tex]y^2 = 3z^2[/tex]. This gives us two sets of equations:

[tex]x^2 + y^2 = z^2 and x^2 + y^2 + z^2 = 25:[/tex]

Substituting [tex]x^2 + y^2 = z^2[/tex] into[tex]x^2 + y^2 + z^2 = 25[/tex], we get [tex]2z^2 = 25[/tex],

which gives z = ±5/√2.

[tex]x^2 + y^2 = 3z^2 and x^2 + y^2 + z^2 = 25:[/tex]

Substituting[tex]x^2 + y^2 = 3z^2[/tex]into [tex]x^2 + y^2 + z^2 = 25[/tex], we get [tex]4z^2 = 25[/tex],

which gives z = ±5/2.

So we have four intersection points: (±5/√2, ±5/√2, ±5/√2) and (±5/2, ±5/2, ±5/2√3).

Find the part of the ball that lies between the cones.

To find the volume of the part of the ball that lies between the cones, we

need to integrate the volume element dV = dx dy dz over the region

enclosed by the cones and the sphere. Since the region is symmetric

about the z-axis, we can integrate over a quarter of the region and

multiply the result by 4.

for such more question on integration

https://brainly.com/question/22008756

#SPJ11

Question

Express the volume of the part of the ball that lies between two cones: one with a half-aperture angle of π/4 and the other with a half-aperture angle of π/3.

Please help it would be amazing if you knew this

Answers

Answer: 5x + 5

Step-by-step explanation:

You combine the two functions togther, and add the like terms.

2x +3 +3x +2

Please give brainliest, have a great night!

Farmer John is building a new pig sty for his wife on the side of his barn.   The area that can be enclosed is modeled by the function A(x) = - 4x^2 + 120x, where x is the width of the sty in meters and A(x) is the area in square meters.   

What is the MAXIMUM area that can be enclosed?

Answers

the MAXIMUM area that can be enclosed is 900 m²

To find the maximum area that can be enclosed, we need to find the vertex of the parabolic function A(x) = -4x^2 + 120x. The vertex represents the maximum point on the parabola.

The x-coordinate of the vertex can be found using the formula x = -b/2a, where a is the coefficient of the x^2 term and b is the coefficient of the x term. In this case, a = -4 and b = 120, so x = -120/(2*(-4)) = 15.

To find the y-coordinate of the vertex, we can substitute x = 15 into the function: A(15) = -4(15)^2 + 120(15) = 900. Therefore, the maximum area that can be enclosed is 900 square meters.

Learn more about maximum area at https://brainly.com/question/9602349

#SPJ11

Emily has 6 pages of homework to do. If she can finish 38 of a page in one hour, how many hours will her homework take?

Answers

Emily's homework will take approximately 9 hours to complete.

Emily has 6 pages of homework and she can finish 3/8 of a page in one hour, we can calculate the total number of hours required to complete her homework.

To find the number of hours, we divide the total number of pages by the number of pages she can finish in one hour:

Number of hours = Total number of pages / Pages finished in one hour

Number of hours = 6 pages / (3/8) pages per hour

To divide by a fraction, we can multiply by its reciprocal:

Number of hours = 6 pages * (8/3) pages per hour

Simplifying the multiplication:

Number of hours = 48/3

Number of hours = 16

Therefore, Emily's homework will take approximately 16 hours to complete.

Hence, the answer is that Emily's homework will take approximately 9 hours to complete.

To know more about fraction , refer here:

https://brainly.com/question/10354322#

#SPJ11

A sheet of paper 82 cm-by-88 cm is made into an open box (i.e. there's no top), by cutting X-cm squares out of each corner and folding up the sides. Find the value of x that maximizes the volume of the box. Give your answer in the simplified radical form. X= is the max.

Answers

The value of x that maximizes the volume of the box is x=11 cm.

Let x be the length of the side of each square cut from the corners of the paper.

The height of the box will be x cm, and the length and width of the base of the box will be (88-2x) cm and (82-2x) cm, respectively.

The volume of the box is given by V(x) = x(88-2x)(82-2x).

Expanding this expression and simplifying, we get V(x) = 4x^3 - 340x^2 + 7040x.

To find the maximum volume, we take the derivative of V(x) with respect to x and set it equal to 0. We get dV/dx = 12x^2 - 680x + 7040 = 0.

Solving this quadratic equation using the quadratic formula, we get x = (680 ± sqrt(680^2 - 4127040))/(2*12).

Simplifying this expression, we get x = (680 ± 120)/24.

Therefore, the two possible values of x are x = 25/3 cm and x = 11 cm.

To determine which value of x maximizes the volume of the box, we evaluate V(x) at both values of x and compare them. We find that V(25/3) ≈ 5757.04 cm^3 and V(11) = 5808 cm^3.

Therefore, the value of x that maximizes the volume of the box is x = 11 cm.

For more questions like Equation click the link below:

https://brainly.com/question/29657983

#SPJ11

An investor purchases 500 shares of Exxon-mobil stock at $98. 93 per share. His broker charges 2% of the cost of the stock. What is the cost of the stock?

Answers

The cost of the stock, including the broker's fee, is $50,454.30.

How to find the total cost of stock?

The cost of the stock can be found by multiplying the number of shares purchased by the price per share. In this case, the investor purchased 500 shares of Exxon-mobil stock at $98.93 per share, so the cost of the stock can be calculated as follows:

Cost of stock = Number of shares × Price per share

Cost of stock = 500 × $98.93

Cost of stock = $49,465

However, the broker charges 2% of the cost of the stock, which is an additional fee that needs to be added to the total cost. To find the broker's fee, we can simply multiply the cost of the stock by 2%:

Broker's fee = 2% × Cost of stock

Broker's fee = 2% × $49,465

Broker's fee = $989.30

Therefore, the total cost of the stock, including the broker's fee, is:

Total cost of stock = Cost of stock + Broker's fee

Total cost of stock = $49,465 + $989.30

Total cost of stock = $50,454.30

Learn more about stocks

brainly.com/question/28663581

#SPJ11

You spin a spinner that has 12 equal-sized sections numbered 1 to 12. Find the probability of p(less than 5 or greater than 9)

Answers

The probability of getting a number less than 5 or greater than 9 is:

P = 0.583

How to find the probability for the given event?

The probability is equal to the quotient between the number of outcomes for the given event and the total number of outcomes.

The numbers that are less than 5 or greater than 9 are:

{1, 2, 3, 4, 10, 11, 12}

So 7 out of the total of 12 outcomes make the event true, then the probability we want to get is the quotient between these numbers:

P = 7/12 = 0.583

LEarn more about probability at:

https://brainly.com/question/25870256

#SPJ1

The time of a pendulum varies as the square root of its length. If the length of a pendulum which beats 15 seconds is 9 cm. Find

(A) the length that beats 80 seconds

(B)the time of a pendulum with length 36 cm

Answers

(A) The length that beats 80 seconds is 256 cm.

(B) The time of a pendulum with length 36 cm is 30 seconds.



(A) According to the given information, the time of a pendulum varies as the square root of its length. Let's denote time as T and length as L. Therefore, T ∝ √L. To find the constant of proportionality, we can use the provided data: T1 = 15 seconds and L1 = 9 cm. So, we have T1 / √L1 = k, where k is the constant. Now, let's find k: k = 15 / √9 = 15 / 3 = 5.

Now, we want to find the length (L2) of a pendulum that beats 80 seconds (T2). We can use the formula T2 = k * √L2. Substituting the values, we get 80 = 5 * √L2. To find L2, we can rearrange and solve for it: L2 = (80 / 5)² = 16² = 256 cm.

(B) To find the time (T3) of a pendulum with a length of 36 cm (L3), we can use the same formula with the known constant k: T3 = k * √L3. Substituting the values, we get T3 = 5 * √36 = 5 * 6 = 30 seconds.

In conclusion, the length of a pendulum that beats 80 seconds is 256 cm, and the time of a pendulum with a length of 36 cm is 30 seconds.

To know more about pendulum, refer to the link below:

https://brainly.com/question/29268528#

#SPJ11

need this asap please

Answers

b. <2 ≅ < 3; corresponding angles are equal

d. < 1 + < 2 = 180 degrees; sum of angles on a straight line

How to determine the reasons

To determine the reasons, we need to know about transversals

Transversals are lines that passes through two lines at the given plane in two distinct points.

It intersects two parallel lines

It is important to note the following;

The sum of the angles on a straight line is 180 degreesAngles at right angle is 90 degreesCorresponding angles are equalAdjacent angles are equal

Learn more about transversals at: https://brainly.com/question/24607467

#SPJ1

O is the center of the regular hexagon below. Find its area. Round to the nearest tenth if necessary

Answers

The area of the regular hexagon is 509.2 square units (to the nearest tenth).

The formula for the area of a regular polygon is:

[tex]\boxed{\text{Area}=\frac{\text{r}^2\text{n sin}\huge \text(\frac{360^\circ}{\text{n}}\huge \text) }{y} }[/tex]

where:

r is the radius (the distance from the center to a vertex).n is the number of sides.

From inspection of the given regular polygon:

r = 14 unitsn = 6

Substitute the values into the formula and solve for area:

[tex]\text{Area}=\dfrac{14^2\times6\times\text{sin}\huge \text(\frac{360^\circ}{6}\huge \text) }{2}[/tex]

       

        [tex]=\dfrac{196\times6\times\text{sin} (60^\circ)}{2}[/tex]

        [tex]=\dfrac{1176\times\frac{\sqrt{3} }{2} }{2}[/tex]

        [tex]=\dfrac{588\sqrt{3} }{2}[/tex]

        [tex]=294\sqrt{3}[/tex]

        [tex]=509.2 \ \text{square units (nearest tenth)}[/tex]

Therefore, the area of the regular hexagon is 509.2 square units (to the nearest tenth).

Here is another one sorry there will be a lot

Answers

Answer:

2 7/24 gallons

(sorry if its wrong)

1. The Daily Statesman newspaper costs $6. 00 per week. The newspaper currently has 700


subscribers. The newspaper wants to increase its revenue and estimates that it will lose 40


customers for every $0. 75 increase in price. What weekly subscription price will maximize the


newspaper's weekly income? Round the answer to the nearest hundredth.

Answers

The newspaper should increase its subscription price by $2.19 to maximize its weekly income and the new subscription price would be  $8.19 per week.

To maximize the newspaper's income, we need to find the price that will result in the highest revenue. Let's assume that the newspaper increases the subscription price by x dollars.

Then the revenue R(x) can be expressed as:
R(x) = (700 - 40x) * (6 + 0.75x)

Expanding the expression, we get:

R(x) = 4200 + 1050x - 240x^2

To find the price that maximizes revenue, we need to find the value of x that maximizes R(x). We can do this by taking the derivative of R(x) with respect to x and setting it equal to 0:

dR/dx = 1050 - 480x = 0

Solving for x,

x = 1050/480 = 2.1875

Therefore, the newspaper should increase its subscription price by $2.19 to maximize its weekly income. The new subscription price would be:

6 + 2.19 = $8.19 per week.

To learn more about income : https://brainly.com/question/28390284

#SPJ11

An arithmetic sequence K starts 4,13. Explain how would you calculate the value of the 5,000th term

Answers

The value of the [tex]5000^{th}[/tex] term in the given arithmetic sequence K is 44995.

The sequence that is given in the question is said to be an arithmetic sequence which means the consecutive elements in the series will have common differences.

To find any term in the series first, we need to find the first term and the common difference that the series follows.

Here we know that the first and the second term of the series are 4 and 13 so from this we can find the common difference which is:

13-4=9

so the first term (a) = 4

the common difference (d) = 9

To find the [tex]n^{th}[/tex] term of the series we can use the formula:

[tex]a_n=a_1+(n-1)*d[/tex]

where [tex]a_n[/tex] is the nth term in the sequence, [tex]a_1[/tex] is the first term of the series, n is the no.of term, and d is the common difference.

So to find the 5000th term in the series

[tex]a_{5000}=4+(5000-1)*9\\a_{5000}=4+(4999*9)\\a_{5000}=4+ 44991\\a_{5000}= 44995\\[/tex]

The value of the [tex]5000^{th}[/tex] term is 44995

Learn more about the arithmetic sequence at:

https://brainly.com/question/28354530

#SPJ4

Hillary used her credit card to buy a $804 laptop, which she paid off by making identical monthly payments for two and a half years. Over the six years that she kept the laptop, it cost her an average of $0. 27 of electricity per day. Hillary's credit card has an APR of 11. 27%, compounded monthly, and she made no other purchases with her credit card until she had paid off the laptop. What percentage of the lifetime cost of the laptop was interest? Assume that there were two leap years over the period that Hillary kept the laptop and round all dollar values to the nearest cent)​

Answers

14.33% of the lifetime cost of Hillary's laptop was interest.

Since Hillary paid off her laptop in two and a half years, and kept it for six years, we need to calculate the compound interest over six years. Accounting for two leap years, there were 365 * 6 + 2 = 2192 days over the period that Hillary kept the laptop. Therefore, the total cost of electricity over that period was 2192 * 0.27 = $592.64.

Plugging in the values, we get:

A = 804 * (1 + 0.1127/12)³⁰= 1003.94

Hillary paid $1003.94 for her laptop, including interest. Subtracting the original cost of the laptop, we get:

Interest = 1003.94 - 804 = 199.94

So Hillary paid $199.94 in interest on her credit card over two and a half years. To calculate what percentage of the lifetime cost of the laptop was interest, we need to divide the interest paid by the total cost of the laptop and electricity:

Lifetime cost = 804 + 592.64 = 1396.64

Percentage of lifetime cost that was interest = (199.94 / 1396.64) * 100% = 14.33%

To know more about compound interest here

https://brainly.com/question/29335425

#SPJ4

A coin (H: heads; T: tails) is flipped and a number cube (1, 2, 3, 4, 5, 6) is rolled. What is the sample space for this experiment?

Answers

The sample space for this experiment contains a total of 12 possible outcomes.

How to find the probability and determine the sample space?

The sample space for this experiment is the set of all possible outcomes. In this case, we have two independent events: flipping a coin and rolling a number cube.

The possible outcomes for flipping a coin are H (heads) and T (tails).

The possible outcomes for rolling a number cube are 1, 2, 3, 4, 5, and 6.

To determine the sample space for the experiment, we need to consider all possible combinations of these outcomes. Therefore, the sample space consists of all possible pairs of outcomes:

Sample space = {(H, 1), (H, 2), (H, 3), (H, 4), (H, 5), (H, 6), (T, 1), (T, 2), (T, 3), (T, 4), (T, 5), (T, 6)}

So the sample space for this experiment contains a total of 12 possible outcomes.

Learn more about probability and sample space.

brainly.com/question/11666439

#SPJ11

A woman claims to have the ability to recognize by tasting it, whether tea was poured first and milk added after, or whether tea was added to milk. In order to test her powers, a set of 10 cups is brought to her and she is asked to taste them. She gets 7 out of 10 correct. Assuming each trial is independent, what is the probability that she would have done at least this well if she had no ability to recognize such difference

Answers

The probability that the woman would have done at least as well if she had no ability to recognize: the difference between the two methods is 0.117.

Let's assume that the woman has no ability to recognize the difference between the two methods. In that case, the probability of guessing the correct answer for each trial is 0.5 (since there are only two options).

The number of correct answers in 10 trials follows a binomial distribution with parameters n = 10 and p = 0.5. We want to calculate the probability of getting at least 7 correct answers.

Using a binomial distribution calculator or a standard normal distribution table, we can find that the probability of getting 7 or more correct answers is 0.117 (rounded to three decimal places).

Therefore, if the woman had no ability to recognize the difference between the two methods, there would still be a 0.117 probability that she would have gotten at least 7 correct answers by chance. Since 0.117 is not a small probability, we cannot reject the null hypothesis that the woman has no ability to recognize the difference between the two methods based solely on this experiment.

To know more about probability, refer here:

https://brainly.com/question/16383548#

#SPJ11

A ship sailed from Port X to Port Y. It traveled 20 kilometers due north and then 25 kilometers due west. If the ship then sailed back using the shortest route, what would the total distance traveled be? Round to the nearest kilometer.

Answers

The total distance traveled by the ship, including the trip from Port X to Port Y and the return trip, is approximately 42 kilometers.

What is Kilometer ?

Kilometer (km) is a metric unit of length or distance, commonly used in many countries around the world. It is equal to 1000 meters, or approximately 0.62 miles.

To find the shortest route back to Port X from Port Y, the ship needs to sail in a straight line. This means that it needs to sail due south for 20 kilometers and then due east for 25 kilometers.

We can now use the Pythagorean theorem to find the total distance traveled by the ship:

total distance = √(400+ 625 + 400+ 625)

total distance = √(1200 + 625)

total distance = √1825

total distance ≈ 42.73 kilometers (rounded to the nearest kilometer)

Therefore, the total distance traveled by the ship, including the trip from Port X to Port Y and the return trip, is approximately 42 kilometers.

To learn more about Kilometer from given link.

https://brainly.com/question/22040991

#SPJ1

please please please please please please help me this is all due tomorrow​

Answers

For the following probabilities:

7. Theoretically, blue will occur 100 times.8. Based on experiment, blue will occur 95-105 times.9. a) 1/4, b) 1/2, c) 3/4.10. a) 0.25, b) 0.5, c) 0.75.11. spade can occur 125 times theoretically.12. experimentally spade occurs 500 times.

How to determine probability?

7. Theoretically, if the spinner is spun 400 times, you would expect to get blue 100 times since blue has a probability of 1/4 or 25% of being selected on each spin.

8. Based on the experiment, if the spinner is spun 400 times, you would expect to get blue around 95-105 times, depending on the margin of error in the experiment. This is based on the observed experimental probability of blue being selected in the given number of spins.

9. a) P(club) = 13/52 or 1/4

b) P(red card) = 26/52 or 1/2

c) P(not a heart) = 39/52 or 3/4

10. a) P(club) = 5/30 or 1/6 in the experiment, which is close to the theoretical probability of 1/4 or 0.25.

b) P(red card) = 13/30 in the experiment, which is close to the theoretical probability of 1/2 or 0.5.

c) P(not a heart) = 27/30 in the experiment, which is close to the theoretical probability of 3/4 or 0.75.

11. Theoretically, if a card is drawn at random 500 times, you would expect to get a spade around 125 times since spades have a probability of 1/4 or 25% of being selected on each draw.

12. Based on the experiment, if a card is drawn at random 500 times, you would expect to get a spade around 110-140 times, depending on the margin of error in the experiment. This is based on the observed experimental probability of spades being selected in the given number of draws.

Find out more on probability here: https://brainly.com/question/251701

#SPJ1

Image transcribed:

7. Theoretically, if the spinner is spun 400 times, how many times would you expect to get blue?

8. Based on the experiment, if the spinner is spun 400 times, how many times would you expect to get blue?

9. A card is drawn from a standard deck of cards. Find each probability.

a) P(club)

b) P(red card)

c) P(not a heart)

10. The table below shows the results of an experiment in which a card was drawn at random 30 times. Find each probability based on the experiment and compare to the theoretical probability.

Result | Frequency

Heart | 3

Diamond | 10

Club | 5

Spade | 12

a) P(club)

b) P(red card)

c) P(not a heart)

11. Theoretically, if a card is drawn at random 500 times, how many times would you expect to get a spade?

12. Based on the experiment, if a card is drawn at random 500 times. how many times would you expect to get a spade?

5-|p+6|=-8

2 answers

NOT 19

Answers

5-|p+6|=-8
-|p+6|=-13
|p+6|=13
-> two equations —-
p+6=13
p+6=-13
(Absolute value means that no matter what is in it it will be positive thus if it equals +-13 it is a valid solution)
—-p=7,-19

Nicole has 28 nickels and dimes that amount to $1. 85 how many of each coin does she have

Answers

Answer:

Nicole has 9 dimes and 19 nickels.

Nicole has 9 dimes and 19 nickels.

Just the answer is fine:)
Let S be the surface in R3 that lies on C = {(x, y, z) ER3 | 22 = 100(x2 + y²)} - and between the planes given by z= 1 and 2 = 5. Then the area of Sis = A(S) Check

Answers

The area of S is:

[tex]A(S) = 16\pi \sqrt{(500/11)}= 128.8[/tex]

How to find the area of S?

The surface S can be described in terms of cylindrical coordinates by setting:

x = r cos(θ)

y = r sin(θ)

z = z

Using these coordinates, we can rewrite the equation for C as:

r² = 22/100(x² + y²) = 22/100r²

Simplifying this equation, we get:

[tex]r = \sqrt{(500/11)}[/tex]

Thus, the surface S is the portion of the cylinder of radius [tex]\sqrt{(500/11)}[/tex] between z = 1 and z = 5.

To calculate the area of S, we can use the formula:

A(S) = ∫∫∂S ||n|| [tex]dA[/tex]

where ||n|| is the magnitude of the normal vector to the surface, and [tex]dA[/tex] is the area element on the surface.

For the cylinder, the normal vector is simply the radial unit vector pointing outward from the origin:

n = (cos(θ), sin(θ), 0)

The magnitude of the normal vector is ||n|| = 1, so we can simplify the formula for the area to:

A(S) = ∫∫∂S [tex]dA[/tex]

To evaluate this integral, we need to parameterize the surface S. We can use the cylindrical coordinates we defined earlier:

x = r cos(θ)

y = r sin(θ)

z = z

with 0 ≤ θ ≤ 2π and 1 ≤ z ≤ 5.

The area element in cylindrical coordinates is given by:

[tex]dA = r \ dz\ d\theta[/tex]

Substituting in our parameterization of S, we get:

A(S) = ∫∫∂S r [tex]dz[/tex] dθ

[tex]= \int\limits^{2\pi }_0 \int\limits^5_1 {\sqrt{(500/11)} dz d\theta}\\= \sqrt{(500/11)} \int\limits^{2\pi }_0 {(5 - 1) d\theta}\\= 16\pi \sqrt{(500/11)[/tex]

Therefore, the area of S is:

[tex]A(S) = 16\pi \sqrt{(500/11)}= 128.8[/tex]

Learn more about surface area of a cylinder

brainly.com/question/22074027

#SPJ11

Other Questions
For never was a story of more woe than of Juliet and her Romeo these lines are written in what meter? Sam has a cylindrical storage container 7 inches tall with a radius of 5 inches. How much cat litter will fit in the container?Use 3. 14 for . Round your answer to the nearest tenth. Answer: _____________ cubic inches Verify the given linear approximation at a = 0. Then use a graphing calculator or computer to determine the values of x for which the linear approximation is accurate to within 0.1. (Round your answers to three decimal places. Enter your answer using interval notation.)sin1(x) x The Maclaurin series for a function f is given by f(x)=xx^3/3!+x^5/5!x^7/7!++(1)^n*x^2n+1/(2n+1)!+ and converges to f(x) for all x. Let g be the function defined by g(x)=f(x2) A rectangular pyramid has a volume of 480 in. If a rectangular prism has a base and height congruent to the pyramid, whatis the volume of the prism? ( point)Please help!!! Quality-control research determined that of all newcars sold by Sherman Motors, 8% will require aminor repair during the first year of ownership. Suppose you survey the owners of three cars fromSherman Motors. Find the probability to the nearestpercent that exactly one car will require a minorrepair in the first year Coxey's Army was a response to following the . Its leader, Jacob Coxey, wanted the federal government to What is the central idea of the passage?A Killer Whales are endangered and need our protection. B Killer Whales are in the dolphin family. C Killer Whales are popular icons in social media. D Killer Whales are on the decline in recent years. What is the total number of moles, to the nearest tenth, of solute contained in 0. 50 liter of 3. 0 M HCl? PLEASE HELP ASAP WILL GIVE 100 POINTS!!!!Suppose a packet that is transmitted across the internet contains the following information (from left to right):Bits 1-4: Packet sequence number within the message.Bits 5-8: Total number of packets in the message.Bits 9-16: Number identifying the sender.Bits 17-24: Number identifying the receiver.Bits 25-64: Part of the actual message being sent.Here is one of the packets being sent over the internet:01111011 10000001 11001110 01010110 00111100 10011100 11100010 10001111Which of the following statements about this packet is true? Select one answerA)This is packet 1 out of 8 total packets in the message.B)This is packet 7 out of 11 total packets in the message.C)This is packet 14 out of 22 total packets in the message.D)This is packet 123 out of 129 total packets in the message. Can someone help me please Amandas new job has a 50% employer match on the first 4% of her salary contributed to her 401(k). How much was deposited in Amandas account if she makes $61,000 a year and she took advantage of the entire contribution match? The frequency table below shows a students quiz scores. One data value in the table is missing. If the mean of the data set is 2.2, what is the score for the missing data item? A. 0 B. 1 C. 2 D. 3 11Please select the best answer from the choices providedOAB1When independent kingdoms were formed, they all competed for theA. food supplyB. trade marketsC. natural resourcesD. protection of the godsD2345 How many moles of chnhcl need to be added to 200.0 ml of a 0.500 m solution of chnh (kb for chnh is 4.4 10) to make a buffer with a ph of 11? Llungby AB spent 1,000,000 krone in 2020 on the development of a new product. The company determined that 25 percent of this amount was incurred after the criteria in IAS 36 for capitalization as an intangible asset had been met. The newly developed product is brought to market in January 2021 and is expected to generate sales revenue for five years. Assume that Llungby AB is a foreign company using IFRS and is owned by a company using U. S. GAAP. Thus, IFRS balances must be converted to U. S. GAAP to prepare consolidated financial statements. Ignore income taxes. Required: Prepare journal entries for development costs for the years ending December 31, 2020, and December 31, 2021, under (1) IFRS and (2) U. S. GAAP. Prepare the entry(ies) that the U. S. Parent would make on the December 31, 2020, and December 31, 2021, conversion worksheets to convert IFRS balances to U. S. GAAP Think about prime numbers and composite numbers.list all the digits that are the last digit of at least one prime number. 3. Which of the following describes how Henry Ward Beecher attempted to support the antislavery cause in Kansas?A. Beecher petitioned to the president and tried to encourage him to bring the conflict to an end.B. Beecher wrote a passionate plea to the people of Kansas, which was reprinted in newspapers across the country.C. Beecher introduced a bill to Congress which, if it had passed, would have made slavery illegal.D. Beecher raised funds so that rifles could be purchased and sent to the antislavery supporters. write a program in Python to calculate a year DIARY ENTRIES 291 Your best friend has informed you that he/she will be visiting you on your birthday to give you a surprise gift. Write TWO diary entries. The first entry must indicate how you felt BEFORE your friend's visit and the second entry must express how you felt AFTER your friend's visit.